Mối quan hệ giữa QMA và AM là gì?


12

Tôi đọc trong SP Jordan, D. Gosset, PJ Love " vấn đề -complete cho Hamilton stoquastic và ma trận MarkovQMMột " mà nó không chắc rằng .QMMộtMộtM

Tôi đã ngạc nhiên về khẳng định này. Vậy mối quan hệ đúng đắn giữa A M là gì?QMMộtMộtM


@Kaveh, chỉnh sửa tiêu đề của bạn không chính xác. Từ "stoquastic" được đánh vần đúng cách. Sự nhầm lẫn tương tự cũng xảy ra trong các bình luận của cstheory.stackexchange.com/questions/3161/iêu
Alessandro Cosentino

1
@Alessandro Cosentino: Tôi đã thay đổi nó trở lại thành stoquastic, cảm ơn.
Kaveh

Câu trả lời:


22

Không có mối quan hệ nào được biết là giữ giữa QMA và AM, và điều hợp lý là phỏng đoán chúng là không thể so sánh được.

Nếu QMA được chứng minh là có trong AM, thì đó sẽ là một kết quả hoàn toàn to lớn về độ phức tạp lượng tử. Tất nhiên, điều đó có nghĩa là BQP nằm trong PH, bản thân nó sẽ rất lớn, nhưng nó sẽ vượt xa điều đó - nó chắc chắn sẽ đòi hỏi những tiết lộ lớn về cấu trúc của thuật toán lượng tử và chứng chỉ lượng tử.

Có nói rằng, bằng chứng chống lại là không thuyết phục. Một lời tiên tri liên quan đến QMA không có trong AM sẽ giúp ích và có vẻ như kết quả như vậy có thể không còn xa - nhưng chúng ta thậm chí còn chưa có điều này.

Một bằng chứng về ngăn chặn ngược, AM trong QMA, cũng sẽ rất lớn. Ít nhất ở đây chúng ta có một lời tiên tri liên quan đến AM không có trong QMA (và trên thực tế thậm chí không được chứa trong PP).


BQP có trong QMA không? Tôi hỏi bởi vì tương đương "cổ điển" (BPP so với NP) hoàn toàn không được biết. (đây là từ việc tôi đọc bình luận của bạn "nó sẽ ám chỉ rằng BQP đang ở PH"
Suresh Venkat

5
@Suresh: Vâng, đúng vậy. BQP và QMA có cùng mối quan hệ với P và NP, hoặc BPP và MA. Trong ba ví dụ này, lớp thứ nhất là tầm thường trong lớp thứ hai, bởi vì lớp thứ hai được định nghĩa là lớp thứ nhất có quyền truy cập vào một "chứng chỉ" hoặc "bằng chứng" có kích thước đa thức.
Robin Kothari

đúng rồi bởi vì cả BQP và QMA đều có một yếu tố ngẫu nhiên, không giống như BPP và NP (cf: câu hỏi khác này về mối quan hệ giữa QMA và NP: cstheory.stackexchange.com/questions/1443/under Hiểu-qma )
Suresh Venkat

12

Chỉ một điều để thêm vào câu trả lời của John:

Theo giả thuyết derandomization hợp lý, AM = NP. Trong trường hợp đó, chắc chắn chúng ta sẽ có AM ⊆ QMA.

Khi sử dụng trang web của chúng tôi, bạn xác nhận rằng bạn đã đọc và hiểu Chính sách cookieChính sách bảo mật của chúng tôi.
Licensed under cc by-sa 3.0 with attribution required.